LSAT and Law School Admissions Forum

Get expert LSAT preparation and law school admissions advice from PowerScore Test Preparation.

 Jacques Lamothe
PowerScore Staff
  • PowerScore Staff
  • Posts: 50
  • Joined: Sep 24, 2013
|
#13045
Hey Anahi,

H and J cannot be drivers because the rules specify who can drive when they are in the car. If H is in a car, F or G drives and if J is in a car, F or k drives. So it is impossible for either to be in a car in which one of them drives.

Could you tell me what templates you are looking at? I am looking at the Powerscore Test 1 explanations and I don't see any templates drawn in the main diagram section or for question 2 specifically. Are you asking about the numerical distributions above the main diagram?

Jordan
 Jkjones3789
  • Posts: 89
  • Joined: Mar 12, 2014
|
#16600
Hello, I was having some difficulty with this game. I am confused as to what either or means. I know what either or but not both means but when I do not see the but not both I get confused because I am not certain if both conditions could exist together. With this game I found the numerical distributions 4-2 and 3-3, being that each car needed at least passengers. However I was confused about the rules. I tried to have care 1 and car 2 with the dual options of F/G on car 1 and K/F or car 2, but that didn't help. I also know that GH can go wherever as long as they are together or at least that is what I thought. Please help me !!! Thank you
 Nicholas Bruno
PowerScore Staff
  • PowerScore Staff
  • Posts: 62
  • Joined: Sep 27, 2011
|
#16606
Let me know if this answers all of your questions. I think you are asking about either/or and how to diagram the game but if you have more questions, let me know!

1) Yes! Either/or means that the two cannot be together. In context of this game, it makes total sense to read it "but not": How can both F and G drive, for instance, with H when we can only have one driver?

2) I set it up like a group game:

H :arrow: F (sub D for drive) OR G (sub D)
J :arrow: F (sub D) OR K (sub D)
G :dbl: L
 srcline@noctrl.edu
  • Posts: 243
  • Joined: Oct 16, 2015
|
#21288
Hello

I also had problems with this game, specifically with the diagramming of the rules and the setup.


Car 1 Car 2

(vertical)
Rules 1 :
F/G(d) :arrow: H

Rule 2:
F/K(d) :arrow: J

Rule 3:
GL (block)

I was also wondering why this is not a vertical set up because of the last rule. Is this not a block then?

Thankyou
Sarah
 Robert Carroll
PowerScore Staff
  • PowerScore Staff
  • Posts: 1787
  • Joined: Dec 06, 2013
|
#21322
Sarah,

It's good to focus on getting the setup right, because so much confusion can come from a faulty setup in this game. Your first choice will be whether you want the 2 cars (which are the base) to be arranged horizontally or vertically. You have them next to each other, which seems great to me. Although the choice is arbitrary, since a horizontal base occurs so often, it's easier to default to that if nothing else helps you decide.

So you'd have the base as you have it.

This isn't a normal grouping game. Although we have multiple people in each car, there is one twist that requires us to track what position some people are in - the existence of a driver. Since each car needs a driver, you'd label one space "Drivers" on the side. I think it makes sense for this space to be the one right above each car number. Above that space, you need another space, as each car has at least two total people, including the driver, but make sure to note that this is a minimum.

Given that, your rules can be diagrammed as blocks, and vertical blocks, as I think you recognized. The conditional representation that you have is not ideal. Instead, the first rule should be a vertical block with "F/G" on the bottom and "H" on top. If this is what you intended, I apologize! It can be hard to represent in text. Similarly, the second rule has "F/K" on the bottom and "J" on top. It makes sense to have the drivers in these rules on the bottom because we labeled the bottom space in each car "driver."

The last rule is block oriented. You'd have a vertical block with "G" over "L" (or vice versa; here we don't intend to imply that either of these is a driver, as we don't know that for sure; we just want to represent the fact that there are in the same car, not that one is a particular passenger in a given car).

I hope this helped!

Robert Carroll
 lsatstudier
  • Posts: 49
  • Joined: Oct 24, 2016
|
#30513
Hi,

Why is Question #4 to this Game C? How would you go about answering this question?

Thank you!
User avatar
 Jonathan Evans
PowerScore Staff
  • PowerScore Staff
  • Posts: 726
  • Joined: Jun 09, 2016
|
#30551
Studier,

On this question, consider first from the restrictions the scenarios in which F is not a driver. Why start here? Because F as Driver is half of an either/or scenario on the necessary side of two conditionals. When F is not a driver, stuff happens, namely G has to be a driver for H and K has to be a driver for J. The thing is though that G also has to be with L. Given that F has to be with exactly two others and that the GD is with H and L, then the other car must have KD, J, and F, leading you to answer C.

Visually, you would implement this information as follows:
  1. Car: GD H __ Car: KD J __
  2. Car 2: GD H L Car 1: KD J F
Notice with this approach we first determine that G and H are together and that K and J are together before we assign L and F. The assignment of F to the car with K and J implies that this will be Car 1.
User avatar
 bnlawyer98
  • Posts: 6
  • Joined: May 27, 2021
|
#87796
How did we know that this a defined game? I was confused because I thought this was an undefined game since it says we need at least two so I thought there could only be 4 people in the cars?
 Robert Carroll
PowerScore Staff
  • PowerScore Staff
  • Posts: 1787
  • Joined: Dec 06, 2013
|
#87823
bn,

We consider this a Defined - Moving game. That's because we know there will be exactly six spaces between the two groups, although it's possible for there to be 3 in each, or 2 in one and 4 in the other. We don't know how those six spaces are allocated, but we know the total sizes of the two groups will always be the same number, which is why this is Defined.

Robert Carroll
User avatar
 bebeg3168
  • Posts: 22
  • Joined: Aug 01, 2022
|
#96492
Hi,
In this LG I assumed F,G,K were the driver and that J, H had to drive separately. If that was the case the rues would have certainly mentioned that J & H had to drive separately correct? My set up was good I think my problem was understanding the rules.

Get the most out of your LSAT Prep Plus subscription.

Analyze and track your performance with our Testing and Analytics Package.